Answer (C) is correct . Unit contribution margin can be used to determine which of the production/sales levels listed produces the highest operating income: ? 1 – 5 6 – 10 11 – 15 16 – 20 Selling price $100,000 $100,000 $100,000 $100,000 Less:? variable costs (50,000) (50,000) (45,000) (45,000) Per unit contr.?margin $??50,000 $??50,000 $??55,000 $?55,000 Times:? unit sales ×???????8 ×??????10 ×??????14 ×??????17 Total contribution margin $400,000 $500,000 $770,000 $935,000 Less:? fixed costs (400,000) (400,000) (600,000) (800,000) Operating income $???????????0 $100,000 $170,000 $135,000
Answer (A) is incorrect because Operating income is $0 at 8 units. Answer (B) is incorrect because Operating income is lower at 10 units than at either 14 or 17 units. Answer (D) is incorrect because Operating income is lower at 17 units than at 14 units.
|